Frage:
Woher wissen wir, dass Quantenverschränkung unabhängig von der Entfernung funktioniert?
Sven Deckers
2018-11-07 19:29:14 UTC
view on stackexchange narkive permalink

Es heißt, dass die Quantenverschränkung unabhängig von der Entfernung funktioniert.2 Partikel können verwickelt werden und Informationen werden sofort ausgetauscht, selbst wenn sie Lichtjahre voneinander entfernt sind.

Aber woher wissen wir, dass dies bei einem so großen Abstand zwischen beiden Partikeln immer noch funktioniert? Ich kann mir Experimente in einem Labor oder sogar auf entgegengesetzten Seiten des Planeten vorstellen, aber nicht mit Lichtjahren dazwischen. Woher wissen wir das?

Verwandte: https://physics.stackexchange.com/q/3158/2451 und Links darin.
"Informationen werden sofort geteilt" Dies wird von vielen QM-Interpretationen völlig abgelehnt und von den meisten zumindest als irreführend angesehen.
Ich habe einen hoch bewerteten Kommentar entfernt, der eine Antwort sein sollte, und ungefähr sechs Antworten darauf.Bitte verwenden Sie Kommentare * nur *, um Verbesserungen für den Beitrag vorzuschlagen, an den sie angehängt sind.
https://physics.stackexchange.com/questions/104050/quantum-entanglement-does-it-necessarily-imply-superluminal-information-transfe/104070#104070
Zehn antworten:
user4552
2018-11-07 20:39:24 UTC
view on stackexchange narkive permalink

Entfernung ist keine relativistische Invariante.Sei A das Ereignis, das durch meinen Beginn markiert ist, um diese Antwort zu schreiben, und B das Ereignis, bei dem ich mit dem Schreiben fertig bin.Im Referenzrahmen meines Schreibtisches ist der Abstand zwischen A und B Null, aber im Referenzrahmen eines Beobachters, der sich mit $ 0.9999999c $ span> relativ zur Erde bewegtbeträgt die Entfernung zwischen A und B Millionen von Kilometern.

Wenn die Trennung zwischen zwei Ereignissen raumartig ist, können wir nur sagen, dass sie in anderen Frames ebenfalls raumartig getrennt sind.Der Abstand zwischen ihnen kann beliebig groß gemacht werden, indem ein geeigneter Referenzrahmen ausgewählt wird.

Wir brauchen also nur ein Experiment, das zeigt, dass die Verschränkung über raumartige Intervalle funktioniert, und daraus folgt, dass sie über beliebig große Entfernungen funktioniert.Ein solches Experiment ist Guerreiro et al., Http://arxiv.org/abs/1204.1712.

In Ihrem ersten Beispiel ist die Trennung zwischen den Ereignissen zeitlich.Und Sie argumentieren, dass der Abstand zwischen ihnen beliebig groß gemacht werden kann, indem Sie sie von einem geeigneten Referenzrahmen aus betrachten.Dieser Teil klingt richtig.Aber wie folgt daraus, dass dies auch für Ereignisse mit raumartiger Trennung gilt?Wird die Längenkontraktion nicht dazu führen, dass es umgekehrt ist, so dass die raumartige Trennung über einen beliebig kleinen Abstand, aber nicht größer erfolgen kann?
Zeit = Raum = Raumzeit.Das erste Beispiel ist raumartig, um das Verständnis zu erleichtern.Der Abstand zwischen A und B ist nicht immer Null.Es ist Null für den Rahmen von Ref auf der Erde, weil er sich mit der gleichen Geschwindigkeit wie der Schreibtisch bewegt und daher die Bewegung der Erde um die Sonne oder der Sonne um die Milchstraße nicht als Faktor in seinen Gleichungen aufzeichnen kann. Die Bewegung des Planeten würde den Abstand von A-B über die Zeit relativ ungleich Null machen. Der Raum für dieselbe Gleichung wird also nicht als absolute Nullbewegung definiert, sondern als relative Bewegung * zum Beobachter *, der sich ebenfalls auf der Erde befindet.
Warte was?Das scheint OP überhaupt nicht zu beantworten, fügen Sie seiner Frage einfach mehr Details hinzu.Guerreiro zeigt eine statistisch signifikante Verteilung in jeder Entfernung, in der er gemessen hat;OP fragt im Wesentlichen: "Wie können wir wissen, dass die Verteilung mit der Entfernung unveränderlich ist?"
@kasperd: Die zeitlichen und raumartigen Fälle sind nicht gleich.Ich habe den zeitlichen Fall nur zur Veranschaulichung der Tatsache diskutiert, dass Entfernungen nicht unveränderlich sind.Für den raumartigen Fall folgt die Tatsache, dass der Abstand beliebig groß gemacht werden kann, durch Aufschreiben der Lorentz-Transformation.Wenn die Ereignisse in einem Frame gleichzeitig auftreten, ist ihre Trennung in einem anderen Frame um einen Faktor $ \ gamma $ größer, der beliebig groß gemacht werden kann.
@fectin: Der Abstand kann beliebig groß gemacht werden, indem einfach der Referenzrahmen geändert wird.
Natürlich kann es.Und wie wirkt sich das auf die Verteilung aus?
Ich argumentiere im Geiste vieler anderer hier veröffentlichter Antworten, dass es bei dieser Frage im Wesentlichen um empirische Tests von Modellen geht.So gesehen reicht diese Antwort nicht aus.Diese Antwort besagt, dass, wenn Sie den Vorhersagen einer speziellen Relativitätstheorie und Quantenmechanik bis zu sehr hohen Geschwindigkeiten glauben, aktuelle Experimente bereits mit der "Arbeit" der Quantenverschränkung über beliebig große Entfernungen übereinstimmen.Aber der Geist dieser Frage bezweifelt die mathematischen Theorien, bis empirische Beweise vorliegen ...
Es gibt noch keine * empirischen * Beweise dafür, dass die Quantenmechanik und die spezielle Relativitätstheorie bei diesen großen Geschwindigkeits- und Entfernungsskalen wie erwartet funktionieren sollten, daher denke ich nicht, dass diese Antwort als Antwort auf die Frage des OP ausreicht.
@jgerber: * Es gibt noch keine empirischen Beweise dafür, dass Quantenmechanik und spezielle Relativitätstheorie bei diesen großen Geschwindigkeits- und Entfernungsskalen wie erwartet funktionieren sollten. * Welche Geschwindigkeiten und Skalen haben Sie im Sinn?Die Lorentz-Invarianz wurde mit unglaublich hoher Präzision bei vielen verschiedenen Geschwindigkeiten und Skalen getestet.Ein Beispiel für Geschwindigkeiten sehr nahe an c wäre das CERN FTL-Neutrino-Debakel, das später gelöst wurde und mit der Lorentz-Invarianz übereinstimmt.
Das ist ein fairer Punkt.Es ist mir nicht klar, dass die Partikelexperimente auf der Art der Verschränkung beruhen, die in Bell-Testexperimenten gemessen wird, und daher ist es möglicherweise nicht fair, die Theorie der Verschränkung auf der Grundlage dieser anderen Experimente auf hohe Geschwindigkeiten zu extrapolieren, obwohl ich meinen Standpunkt zugebeist schwach.Mein zentraler Punkt ist, dass Ihr Beitrag eine theoretische Extrapolation erfordert, die je nach Pedantik des Lesers gerechtfertigt sein kann oder nicht.Oder besser gesagt, je nachdem, wie sehr der Leser aktuelle Beweise für das Ziel der Erweiterung der derzeit akzeptierten Theorien herauspicken möchte.
(1/2) @jgerber: Wir wissen aus Experimenten, dass die Verschränkung viel schneller als die Lichtgeschwindigkeit sein müsste: https://arxiv.org/abs/1303.0614 Und wir wissen aus der Theorie, dass Verschränkungen, die sich mit einer endlichen Geschwindigkeit schneller als $ c $ bewegen, für die superluminale Kommunikation verwendet werden können: https://www.nature.com/articles/nphys2460
(2/2) Ich glaube nicht, dass es im Prinzip möglich ist zu sagen, dass es keine denkbare Theorie gibt, die eine solche superluminale Signalübertragung zulässt, während sie mit bestehenden Experimenten und den Grundbegriffen der Kausalität, die für die physikalische Welt erforderlich sind, kompatibel isthabe eine kohärente Logik.Es ist niemals möglich, definitiv zu sagen, dass die Theorie, die man hat, die einzig richtige Theorie ist.Man kann jedoch mit Recht sagen, dass eine Theorie der Ausbreitung der Verschränkung eine vollständige Überarbeitung der Grundlagen der modernen Physik erfordern würde. Die Beweislast liegt sicherlich bei der Person, die behauptet, dass sie lebensfähig sein kann.
@Rococo "Aber es ist fair zu sagen, dass eine Theorie der Ausbreitung der Verschränkung eine vollständige Überarbeitung der Grundlagen der modernen Physik erfordern würde. Die Beweislast liegt sicherlich bei der Person, die behauptet, dass sie lebensfähig sein kann."Ich stimme dieser Aussage zu.Aber ich füge die Vorsicht hinzu, dass wir Fragen wie die OPs nicht abschreiben sollten, weil sie "das Umschreiben der modernen Physik erfordern".Man kann moderne Bell-Tests als Präzisionstests der Gesetze der Spezialmechanik und der Relativitätstheorie interpretieren.Präzisionstests können, sobald sie extrem genug sind, neue Physik aufdecken.
Vielleicht werden wir feststellen, dass wir, egal wie sehr wir uns bemühen, möglicherweise keine Bell-Korrelationen zwischen einem erdgebundenen Teilchen und einem Teilchen auf dem Mond erkennen können.Dies bedeutet, dass entweder mit der Relativitätstheorie oder der Quantenmechanik etwas nicht stimmt.Vielleicht hilft es zum Beispiel, diese Inkonsistenz zu erforschen, um Theorien der Quantengravitation zu finden.Die Vorsicht ist dann, dass wir weiterhin Fragen wie die OP stellen sollten, oder wir können eine Theorie fälschlicherweise extrapolieren und etwas verpassen.
anna v
2018-11-07 20:38:42 UTC
view on stackexchange narkive permalink

Der Begriff "Quantenverschränkung existiert" ist eine Teilmenge des Begriffs "es existiert eine einzelne quantenmechanische Wellenfunktion, die das System beschreibt".

Wenn Sie die Wellenfunktion kennen, dh eine mathematische Beschreibung haben, hat die Theorie der Quantenmechanik Einschränkungen hinsichtlich der Quantenzahlen und ihrer Erhaltung, so dass Quantenzahlen "verwickelt" sind, weil es eine einzelne Wellenfunktion gibt, die das System als ayc beschreibt gibt auch an.

Nehmen wir ein einfaches Beispiel, den pi0-Zerfall. Es zerfällt in zwei Photonen / Gammas. Der pi0 hat Spin Null. Ausgehend von der Erhaltung der Quantenzahlen (in diesem Fall der Drehimpulserhaltung) in der Abklingwellenfunktion muss eines der Photonen Spin +1 und das andere -1 haben, wobei die Richtung die Richtung ihrer Bewegung ist.

Sobald der Zerfall stattgefunden hat und keine Wechselwirkungen mehr vorliegen, sind die Drehungen für den Experimentator unbekannt, werden jedoch durch die Wellenfunktion definiert.

Wenn ich einen im Labor messe und der andere Alpha Centauri erreicht, kenne ich seinen Spin, indem ich den im Labor gemessen und die Drehimpulserhaltung verwendet habe.

Dies ist Ihre Verwirrung:

und Informationen werden sofort geteilt, auch wenn sie Lichtjahre voneinander entfernt sind

Die Informationen werden nicht von den Photonen geteilt, die Informationen befinden sich im Kopf des Experimentators, der im Labor sitzt.

Woher wissen wir, dass dies wahr ist? Weil wir die quantenmechanische Theorie mit unzähligen Experimenten validiert haben, der Mathematik vertrauen und alle Informationen hier im Labor extrapolieren.

Verschränkung ist mehr als nur eine gemeinsame Wellenfunktion, die das System beschreibt.Der Punkt ist, dass der Zustand nicht als Tensorprodukt über Hilbert-Räume für die Subsysteme zerlegt werden kann.Wenn es geht, gibt es keine Verstrickung.Zum Beispiel ist $ (| \ uparrow \ downarrow \ rangle + | \ downarrow \ uparrow \ rangle) / \ sqrt {2} $ verwickelt, $ | \ uparrow \ downarrow \ rangle $ jedoch nicht.
@J.G.Ich habe ein experimentelles Beispiel gegeben.Die Erhaltung der Quantenzahlen im beobachtbaren Ergebnis gilt unabhängig von der mathematischen Formulierung der Wellenfunktion."Verschränkung" beschreibt eine zusätzliche Einschränkung unter Verwendung von Erhaltungsgesetzen.Wie Wellenfunktionen Erhaltungsgesetze auferlegt werden, ist ein mathematisches Problem.
Der letzte Absatz dieser Antwort hätte etwa zweihundert Jahre nach der Principia über die Newtonsche Mechanik gesagt werden können.
@MartinArgerami.Sicher, nur Messungen führen uns an der Nase zur Quantenmechanik.Wenn neue, genauere (oder was auch immer) Messungen die Quantenmechanik ungültig machen, wird sie anstelle der zugrunde liegenden Schicht aller klassischen Modelle zu einer der Schichten mit ihrem Gültigkeitsbereich.Es ist "Schildkröten den ganzen Weg nach unten".
Aber es gibt immer noch den "gruseligen" Teil, wenn Sie das Experiment so gestalten, dass die Beobachtungsmethode bestimmt, in welche Richtung der Spin beobachtet wird. Bis zu dieser Beobachtung kann die Wellenfunktion anders als das andere Teilchen das Gegenteil von sagenein unbekannter Spin?
@Michael Die Wellenfunktion ist Mathematik und enthält nur Erhaltungsgesetze.Ähnlich verhält es sich mit mechanischen Flugbahnen, die das Auftreffen auf den Boden vorhersagen.Die Wellenfunktion sagt nur Wahrscheinlichkeiten voraus.Das ist der Unterschied.die Wahrscheinlichkeitsvorhersagen, nicht Trajektorien und Massenverteilungen.Es ist nicht die Beobachtungsmethode, jedes korrekte Experiment sollte die gleiche Wahrscheinlichkeitsverteilung ergeben, sonst wird das Modell ungültig.
j4nd3r53n
2018-11-07 21:42:32 UTC
view on stackexchange narkive permalink

Die kurze Antwort lautet "Nein". Tatsächlich ist die grundlegende Tatsache des Lebens in der Wissenschaft, dass wir sozusagen nie wissen: Wir haben eine Theorie, sie macht Vorhersagen, die wir testen, und wenn der Test fehlschlägt, wissen wir mit Sicherheit, dass die Vorhersage falsch war. grob gesagt. Wenn es nicht fehlschlägt, wissen wir nicht wirklich, dass wir Recht hatten - aber wir können uns berechtigt fühlen, zu glauben, dass wir es sind. Selbst wenn wir $ 10 ^ 7 $ span> Mal erlebt haben, dass ein Stein herunterfällt, wissen wir streng genommen immer noch nicht, dass dies beim nächsten Mal der Fall sein wird.

Natürlich würden einige - die meisten - sagen, dass dies unnötige Sophistik ist, und ich würde in den meisten Fällen eher innerhalb angemessener Grenzen zustimmen. Wir haben in Experimenten bestätigt, dass Verschränkung über Entfernungen von mehreren hundert Kilometern zu funktionieren scheint, glaube ich, aber die Behauptung, dass sie über Lichtjahre funktionieren wird, ist immer noch sehr viel Spekulation oder Wunschdenken, IMO.

=== EDIT ===

Angesichts der bisherigen Kommentare denke ich, ich sollte vielleicht meine Position klarstellen. Die grundlegende Erkenntnis, die in der sogenannten wissenschaftlichen Methode formalisiert wird, ist, dass wir niemals durch Experimente absolute Gewissheit über die Gültigkeit der Theorie erlangen können - die einzige absolute Wahrheit kommt in Form von Fälschung; Eine erfolgreiche Theorie ist eine, bei der wir es immer wieder versäumt haben, die Vorhersagen zu verfälschen. Dies ist etwas, das wir immer berücksichtigen müssen, wenn wir basierend auf unserer Theorie extrapolieren - denken Sie nur an Newtons Gravitationstheorie: Sie funktioniert wunderbar für (fast) alle praktischen Zwecke, wir verwenden sie, wenn wir unsere kieselgroßen Satelliten aus und um mehrere schleudern Planeten und schaffen es immer noch, das Ziel meistens zu treffen - aber es konnte die Präzession des Perihels von Merkur nicht vollständig erklären.

Eines Tages - so hofft man bald - werden wir Situationen finden, in denen sowohl GR als auch QM versagen. Dann machen wir den nächsten Schritt nach vorne.

=== EDIT2 ===

Ich sehe, dass meine Antwort abgelehnt wurde - das ist in Ordnung für mich, aber bitte, wenn Sie so denken, fügen Sie einen Kommentar hinzu, damit ich erfahren kann, ob ich einen Fehler gemacht habe, unhöflich war oder irgendetwas anderes.Ich werde es dir nicht vorenthalten.

Ich war bis zum letzten Satz bei dir.Es ist weder.Es ist eine Konsequenz der Theorie, dass es tut.Wir haben festgestellt, dass die Theorie ihre Berechtigung hat.Es ist so wahr wie eine Verschränkung bei genau 2.050421 cm.Ungetestet, aber wir gehen davon aus, dass es so lange dauert, bis wir einen Grund finden, die Theorie zu verbessern.
Ich würde diese Antwort 100 Mal positiv bewerten, wenn ich könnte.Ich würde zustimmen, dass die Extrapolation von mehreren hundert Kilometern auf Lichtjahre (oder Milliarden von Lichtjahren) eine * signifikante * Extrapolation ist, die über die Bedingungen hinausgeht, unter denen die Theorie experimentell validiert wurde, dass * unser Vertrauen in das noch geltende Ergebnis * erheblich * verringert werden sollte.Das bedeutet nicht, dass die Theorie auf größeren Entfernungen falsch ist, nur dass wir es wirklich nicht wissen, da wir auf solchen Entfernungen keine Beweise dafür oder dagegen haben.Und intellektuelle Demut erfordert, dass wir dies anerkennen.
Ich bin froh, Ihnen mitteilen zu können, dass Milliarden von Menschen (einschließlich mir) wissen, dass Steine herunterfallen.Es tut mir leid, dass Sie irgendwann von der Realität abgelenkt wurden und die axiomatische Definition von Wissen akzeptiert haben.Es ist nur in Mathematik und Religion nützlich, nicht aber in den Naturwissenschaften, da ihnen Axiome fehlen.
Ich denke, j4nd3r53n hat tatsächlich darauf hingewiesen, dass wir wissen, dass Steine herunterfallen, aber das ist für einen Fall, in dem die Wahrscheinlichkeit so nahe bei 1 liegt, dass sie vernünftigerweise an 1 angenähert wird. In dem Fall, um den es in der Frage geht, können wir nicht einmalSchätzen Sie die Wahrscheinlichkeit für große Entfernungen, da wir auf diesen Entfernungen noch nie Experimente durchgeführt haben (im Vergleich zu dem Gesteinsexperiment, das wir unglaublich oft durchgeführt haben).Das Beste, was wir tun können, ist, die Wahrscheinlichkeit für kürzere Entfernungen, für die es Experimente gibt, abzuschätzen und dann auf die längeren Entfernungen zu extrapolieren.
"Mit so nahe an 1, dass es ungefähr 1 ist" Ich meine, wir haben (meines Wissens) noch nie einen Stein beobachtet, der * nicht * herunterfällt.So ziemlich jeder würde dies "wissen, dass der Stein fallen wird" nennen, mich eingeschlossen.Aber technisch ist es nicht, zumindest nicht, wenn Sie sich nicht der Philosophie des Empirismus anschließen.
Die Antwort wäre "wir nicht", nicht "nein", da die Frage lautet "woher wissen wir das?"
"Wir alle wissen, dass Steine herunterfallen", bis wir einen unbeschleunigten Rahmen entdecken.
Emilio Pisanty
2018-11-08 17:03:35 UTC
view on stackexchange narkive permalink

Es heißt, dass die Quantenverschränkung unabhängig von der Entfernung funktioniert. 2 Partikel können verwickelt werden und Informationen werden sofort ausgetauscht, auch wenn sie Lichtjahre voneinander entfernt sind.

Um ganz klar zu sein: entanglement kann nicht zur Kommunikation von verwendet werden, egal wie lang oder wie kurz die räumliche Trennung ist. Dies wird in diesem Thread ausführlicher erläutert, aber das Grundprinzip lautet:

  • Angenommen, Sie haben zwei verschränkte Partikel, z. B. Drehungen im Up-Down-Verschränkungszustand $ | {\ uparrow} {\ downarrow} \ rangle + | {\ downarrow} {\ uparrow} \ rangle $ span>.
  • Angenommen, Sie messen das erste Partikel entlang der Auf-Ab-Basis und erhalten das Ergebnis $ s $ span>. Dann wird auch der zweite Spin auf den Status $ - s $ span> projiziert.
  • Abhängig von Ihrer Interpretation des QM und Ihrer allgemeinen Haltung zu seinen grundlegenden Fragen ist es außerdem möglich, dies als sofortige Aktion auf dem zweiten Qubit zu interpretieren, unabhängig von der Entfernung zwischen ihnen.
  • HJedoch: Sie haben keine Kontrolle darüber, ob Sie das Ergebnis $ s = {\ uparrow} $ span> oder erhalten $ s = {\ downarrow} $ span>, sodass Sie keine Kontrolle über die "Nachricht" haben, die gesendet wird.

Dies kann im No-Communication Theorem weiter formalisiert werden, der im Grunde besagt, dass, wenn das System den Regeln der Quantenmechanik folgt, keine Shenanigans verwendet werden können, die Sie möglicherweise in diese Richtung ziehen könnten schneller als Licht zu kommunizieren. Die Quantenmechanik ist eine vollständig kausale Theorie, d. H. Es gibt keine Szenarien, in denen die Auswirkungen einer bestimmten Ursache außerhalb ihres zukünftigen Lichtkegels beobachtet werden können.

Andererseits ist es möglich, die in den obigen Aufzählungspunkten beschriebene Situation so zu interpretieren, dass die Partikel auf nicht kausale Weise "schneller als Licht" miteinander kommunizieren und dann "Verschwörung", um diesen FTL-Kommunikationskanal für makroskopische Experimente nicht verfügbar zu machen - aber die zweite Hälfte dieser Kombination ist entscheidend und kann niemals ausgelassen werden. Einige Leute sind damit einverstanden, aber ich finde es als philosophische Position zutiefst unbefriedigend.

Um ganz klar zu sein, beruht das oben beschriebene Protokoll nicht wirklich auf Verschränkung, und es ist anfällig für eine "Bertlmanns Socken" Erklärung, bei der Sie einfach eine $ \ uparrow $ span> und ein $ \ downarrow $ span> drehen sich in unbeschrifteten Kisten und versenden sie. Diese Art der Erklärung lokaler versteckter Variablen reicht jedoch nicht aus, um den vollständigen Satz von Messergebnissen zu erklären, die unter Verwendung verschränkter Zustände möglich sind: Theorien versteckter Variablen werden durch den Satz von Bell eingeschränkt, um a zu erfüllen Satz von Ungleichungen über die Arten und Mengen von Korrelationen, die sie zeigen können, und es gibt mehrere Experimente, die zeigen, dass quantenmechanische Systeme diese Ungleichungen routinemäßig aufheben.


Mit all diesen Präzisionen:

Es heißt, dass die Quantenverschränkung unabhängig von der Entfernung funktioniert.

Ja, das ist richtig: Soweit wir wissen, funktioniert alles oben Genannte unabhängig von der räumlichen Trennung zwischen den Partikeln.

Aber woher wissen wir, dass dies bei einem so großen Abstand zwischen beiden Partikeln immer noch funktioniert? Ich kann mir Experimente in einem Labor oder sogar auf entgegengesetzten Seiten des Planeten vorstellen, aber nicht mit Lichtjahren dazwischen. Woher wissen wir das?

Wir haben keine Beweise dafür, außer der Tatsache, dass die Theorie funktioniert hat, um jedes Experiment zu erklären, um das wir gebeten haben, auf allen Skalen zu erklären, die wir geschafft haben, Arbeitstests der Theorie zu konstruieren, und die Tatsache, dass alleUnsere astronomischen Beobachtungen der Physik von Orten, die für uns unzugänglich sind (vom Vorhandensein von Helium in der Sonne bis zum thermischen Spektrum des kosmischen Mikrowellenhintergrunds), können mit denselben physikalischen Gesetzen erklärt werden, die wir in erdgebundenen Labors testen.

Es ist durchaus möglich, dass ein bestimmtes physikalisches Gesetz außerhalb des Bereichs bricht, in dem wir es getestet haben. Deshalb testen wir sie weiterhin in neueren und größeren Regimen.in der Tat, wenn wir solche Abweichungen finden, wären sie viel interessanter als die Feststellung, dass es überall gleich ist.Bis und solange wir kein solches Ergebnis finden, gibt es keine Hinweise darauf, dass dies nicht der Fall ist.

Cleonis
2018-11-08 02:18:44 UTC
view on stackexchange narkive permalink

Im Laufe der Zeit gelang es den Experimentatoren, die Entfernung, über die eine Verschränkung nachgewiesen werden konnte, bis zu vielen Kilometern zu erhöhen.

an Lassen Sie mich einen Vergleich mit der Geschichte des Elektromagnetismus anstellen: Als Maxwell die Theorie des Elektromagnetismus veröffentlichte, die wir jetzt als Maxwellsche Gleichungen kennen, war eine der Implikationen der Theorie, dass es möglich sein sollte, eine sich ausbreitende elektromagnetische Welle zu erzeugen und diese elektromagnetische Welle zu empfangen. Heinrich Hertz machte sich darauf, dies zu überprüfen. Der von ihm konstruierte Emitter wurde getreten, um einen sehr kurzen, sehr starken Wechselstrom zu führen, indem er ein Funken übersprang. Der starke Wechselstrom verursachte eine elektromagnetische Welle. Der Empfänger befand sich ziemlich neben dem Sender. Der Empfänger nahm die elektromagnetische Welle auf. Ähnliches gilt für die Quantenverschränkung. Die ersten Demonstrationen waren über relativ kurze Strecken, die Abmessungen eines einzelnen Bankaufbaus. Aber zu Elementarteilchen sind sogar Entfernungen von Zentimetern groß. Also: Sobald die Quantenverschränkung auf einem einzigen Bankaufbau demonstriert wurde, war das Bild ziemlich klar: Nichts in der Theorie, was auf eine Grenzentfernung hindeutet, und eine Demonstration über eine Entfernung von Zentimetern ist bereits eine Demonstration über eine große Entfernung.

Je länger der Abstand, desto geringer ist natürlich die Ausbeute.Je länger die Transitzeit umso größer die Wahrscheinlichkeit ist, die Verflechtung entlang des Weges zu verlieren.Um eine akzeptable Anzahl von Bestätigungen pro Sekunde zu erhalten, müssen Sie größere Populationen von gefangenen Teilchen erzeugen und senden.Wir haben jedoch allen Grund zu der Annahme, dass es keinen Abstand gibt, über den hinaus die Rendite auf Null geht.

Interessanterweise fasste Maxwell alle Gleichungen in einer einzigen Quaternionsform zusammen (Art. 623, IIRC), bei der es sich im Wesentlichen um die 4d-Raum-Zeit-Kontraktion handelt.Es hängt davon ab, ob man glaubt, dass die Metrik an erster Stelle steht oder der 4d-Raum.
Cort Ammon
2018-11-08 05:21:54 UTC
view on stackexchange narkive permalink

Die Antwort lautet "Wir wissen es nie." Es gibt keine Möglichkeit, mit Sicherheit zu wissen, ob die Gesetze der Physik auf der anderen Seite des Jupiter gleich funktionieren. Natürlich haben die Beobachtungen, die wir über Jupiter hinaus gemacht haben, alle die Schlussfolgerung gestützt, dass die Gesetze der Physik überall gleich sind, aber wir werden es nie genau wissen.

Dementsprechend müssen wir nur "empirisch ausreichende" Beweise dafür haben, dass die Effekte auf große Entfernungen wirken. Dafür stellen wir fest, dass Entfernungen wirklich nicht der Hauptbeschränkungsfaktor zu sein scheinen. Die interessantere Grenze ist die Lichtgeschwindigkeit. Wenn Verschränkung dies gut modelliert, ist dies eine nützliche Theorie.

Geschwindigkeit ist einfacher als Distanz. Wenn ich zwei Messungen so einstellen kann, dass sie nur Nanosekunden voneinander entfernt sind, kann mein Experiment in einem Raum stattfinden und dennoch verwicklungsähnliche Effekte aufweisen, die nicht durch Übertragungen erklärt werden können, die sich mit Lichtgeschwindigkeit fortbewegen. Wenn die Messung auf halbem Weg zwischen den Teilen meines Experiments durch ein gemeinsames Signal ausgelöst wird, können wir sehen, dass ich mit einem verwickelten Paar auf eine Weise interagieren kann, die viel schneller ist, als es die Kommunikation mit Lichtgeschwindigkeit zulässt, da keine Zeit für Runden bleibt Ausflüge.

Ist das also ein Beweis dafür, dass die Effekte über die Entfernung von Lichtjahren wirken? Nein. Wenn ich es in einem 10-Fuß-Raum gemacht habe, beweist es nichts darüber, ob die Verschränkung in einem 11-Fuß-Raum funktioniert. Alle unsere bisherigen Experimente haben jedoch gezeigt, dass die Entfernung überhaupt keine Auswirkung hat.

cmaster - reinstate monica
2018-11-08 05:46:36 UTC
view on stackexchange narkive permalink

Leider erinnere ich mich nicht an alle Details, aber es sind die Experimente, die als Reaktion auf das EPR-Paradoxon durchgeführt wurden, die uns gezwungen haben, die Nichtlokalität eines verwickelten Zustands zu akzeptieren.

Diese Experimente erzeugen im Grunde genommen einige Paare verschränkter Teilchen, hauptsächlich Photonen, senden sie in einiger Entfernung voneinander und messen beide auf zufällige Weise. Es kann gezeigt werden, dass die genaue Statistik dieser Messungen erfordert, dass eine Messung tatsächlich die andere beeinflusst. Dieses Ergebnis ist, was die Quantenmechanik vorausgesagt hat und was Einstein, Podolsky und Rosen absurd fanden (weshalb es als Paradox bezeichnet wird). Afaik, die Konsequenz dieser Ergebnisse ist, dass wir entweder die Nichtlokalität verschränkter Zustände akzeptieren müssen oder unsere Vorstellung von der physischen Realität in Frage stellen müssen.

Diese Experimente wurden mit verschiedenen Entfernungen durchgeführt. Ich weiß nicht, wie hoch die aktuelle Rekordentfernung ist. Grundsätzlich ist es jedoch sinnlos, über Entfernungen zu streiten, wenn die beiden Messungen weit genug voneinander entfernt sind, dass das Licht den Ort der anderen Messung nicht erreichen kann, bevor diese Messung tatsächlich stattfindet. Das ist für Photonen leicht zu erreichen: Wenn die verschränkten Photonen in entgegengesetzte Richtungen abgehen und Sie eine Messstation unmittelbar nach einer Entfernung von einem Meter platzieren, finden die Messungen zum gleichen Zeitpunkt im Referenzrahmen des Labors statt, jedoch zu zwei Meter voneinander entfernt. Dies reicht aus, um eine Kommunikation mit Lichtgeschwindigkeit zwischen den verwickelten Partikeln auszuschließen.

Dies könnte von Interesse sein: [Ausweitung von Wheelers Experiment mit verzögerter Auswahl auf den Weltraum] (https://arxiv.org/abs/1704.01911)
Schauen Sie sich beispielsweise Ghost Imaging an, bei dem mithilfe von Zwillingsphotonen Objekte erkannt werden, die von einem Photon gesehen werden, indem der Effekt auf das andere beobachtet wird.https://en.wikipedia.org/wiki/Ghost_imaging Der größte Teil des Problems besteht darin, zu entscheiden, ob wir eine 4d-Raumzeit haben (alle Punkte eindeutig) oder ob die 4d-Messung nur eine Projektion aus einer anderen Dimension ist (z. B. alle Punkte auf)der Lichtkegel ist gleich, wenn er sich in einem sehr kleinen Ursprungsabstand befindet - dem Verschränkungsabstand).
R.. GitHub STOP HELPING ICE
2018-11-09 08:26:15 UTC
view on stackexchange narkive permalink

Die Prämisse, dass "Quantenverschränkung überhaupt funktioniert ", ist falsch - es gibt keine Aktion - ebenso wie die, dass "Informationen überhaupt geteilt werden" (unabhängig davon, ob sie augenblicklich sind oder nicht). . Die Quantenverschränkung ist in keiner Weise ein Vektor für die Übertragung von Informationen, und sie ist wirklich nicht tiefer als zwei Kugeln, eine rote und eine blaue, in eine Schachtel zu legen, ohne sie anzusehen, und die zu trennen Kisten, eine Kiste öffnen, eine rote Kugel sehen und daraus schließen, dass sich in der anderen Kiste eine blaue Kugel befindet.

Als ich die obige Analogie ursprünglich als Kommentar gepostet habe, wurde der Einwand erhoben, dass sie irreführend ist und impliziert, dass die Quantenverschränkung mit (lokalen) versteckten Variablen erklärbar ist. Dies ist natürlich nach Bell's Theorem sowie vielen experimentellen Ergebnissen unmöglich, und die Analogie "nicht tiefer als" sollte nicht so interpretiert werden, dass es eine versteckte Variable gibt. Wenn Sie wirklich wollen, können Sie sich vorstellen, es als Gedankenexperiment im Katzenstil von Schrödinger einzurichten und die Bälle in einem (ungefähr) isolierten Teil des Systems nach einem echten Zufallsprozess zu verteilen, der außerhalb nicht beobachtet wird. Dies ist jedoch nicht der Punkt.

Der Punkt ist, dass es bei der Quantenverschränkung wie in der Ballanalogie darum geht, Schlussfolgerungen aus einer Beziehung zu ziehen, in der unter der Bedingung einer vorherigen Beobachtung des Aufbaus eine Korrelation zwischen bestimmten Observablen zum Ergebnis besteht .

Dies alles ist eine Folge eines allgemeinen Prinzips, dass es im QM wirklich nichts Tieferes gibt als bedingte Wahrscheinlichkeiten für ein Riesenmodell, die bedeutungslos wären, um Vorhersagen ohne Konditionierung auf Beobachtungen zu treffen.

Chris Becke
2018-11-09 19:34:12 UTC
view on stackexchange narkive permalink

Ich betrachte die Frage, inwieweit die Quantenverschränkung so funktioniert:

Solange Verschränkung als mysteriöses Quantenphänomen betrachtet und im Hinblick auf Wellenfunktionen oder andere solche mathematischen Beschreibungen diskutiert wird, ist es schwer, sie als etwas zu betrachten, das große Entfernungen über Raum (und Zeit) hinweg überleben kann.

Wenn Sie jedoch direkt zur Wurzel zurückkehren, haben wir beim Experimentieren mit korrelierten Eigenschaften eine Verschränkung entdeckt. Eigenschaften, von denen wir wissen, dass sie korreliert sind, weil - wenn sie nicht wären - konservierte Größen - wie der Drehimpuls - nicht konserviert würden. Während GR einige Lücken hinterlässt und das Universum Energie erzeugen kann, gibt es im Universum keine Hinweise darauf, dass die Erhaltung streng lokal ist und Energie und Impuls und alle anderen konservierten Eigenschaften trivial gebrochen werden können, wenn Sie dem etwas mehr Platz hinzufügen Mitte des Experiments.

d.h. Jede Entfernung, die wir zeigen können, ist zu kurz, um eine Symmetrieverletzung zu verursachen. Dies ist auch eine Entfernung, über die wir erwarten können, dass die Quantenverschränkung funktioniert.
XXDD
2018-11-10 14:02:39 UTC
view on stackexchange narkive permalink

Persönlich möchte ich sagen, dass die Aussage bisher nicht genau oder zumindest nicht klar ist.Wenn wir allgemein glauben, dass die Struktur der Raumzeit eng mit der Verschränkung zusammenhängt, sollten wir sehr vorsichtig sein, um über 'Entfernung / Zeit / Geschwindigkeit' zu sprechen, wenn es sich um eine Verschränkung handelt.

Meiner Meinung nach scheint ER = EPR zu behaupten, wenn wir mit zwei verschränkten Partikeln spielen, dass die 'normale' Raumzeitstruktur durch die Verschränkung gebrochen (oder zumindest gestört) wird.In dieser Situation ist es nicht sehr streng zu sagen, dass die beiden Partikel weit entfernt sind, da das winzige Wurmloch zwischen ihnen möglicherweise nur eine Abkürzung darstellt, sodass ihr Abstand 0 beträgt. Tatsächlich zeigt die beobachtete Tatsache, dass verschränkte Partikel einen Augenblick zeigenKorrelation scheint zu behaupten, dass sie der gleiche Punkt sind.

Schließlich glaube ich, bevor wir die Quantengravitation zur Integration von QM und GR vervollständigen können, sollte es keine eindeutige Antwort auf diese Frage geben.



Diese Fragen und Antworten wurden automatisch aus der englischen Sprache übersetzt.Der ursprüngliche Inhalt ist auf stackexchange verfügbar. Wir danken ihm für die cc by-sa 4.0-Lizenz, unter der er vertrieben wird.
Loading...